Math, asked by raiamita14, 9 months ago

prove that (Psin theta +Qcos theta)^2+(Pcos theta - Qsin theta)^2=P^2+Q^2

Answers

Answered by kushaal1607
3

Step-by-step explanation:

hope it helps u love u ...

Attachments:
Similar questions